Thread: AOA question
View Single Post
Old 01-30-2011, 09:26 AM
  #1  
698jet
Gets Weekends Off
 
698jet's Avatar
 
Joined APC: Aug 2007
Position: 777er driver
Posts: 101
Default AOA question

would a wing with flaps extended stall at a lower or a higher angle of attack than the same wing with flaps retracted. and if both wings were at the same angle of attack which would produce more lift. take a crack at this question I like to see if I'm getting the same answer as I think my son and his friend have differnt answer then me so I said lets see what other people say.
698jet is offline